2
$\begingroup$

Given a square of size n by m, how many ways could we choose sites, such that all the sites are connected?

By "connected" we mean "connected" by adjacent sites. We will illustrate by example, say, we choose all the "1" sites and all the "0" sites are not chosen. Then

00000
00100
00100
00110
00000

is connected and

00000
00100
00010
00010
00000

is not connected.

A more difficult question would be, how many ways could we choose sites. Such that there are at most 2 connected "island"(or component):

For example:

00001
00001
00110
00100

has 2 connected component and

00001
00100
00010
00000

has 3 connected component.

The ultimate question is, given a cube size of n by m by k, how many ways could we choose sites, such that there are at most j connected components?

I face this question in my research in designing algorithm for material science. Thank you:)

$\endgroup$
8
  • 1
    $\begingroup$ Do you want an exact solution or would some kind of estimates be enough? If the number of possible choices is $N(n,m,k;j)$, then naive arguments show that $\sum_{i=0}^j{nmk\choose i}\leq N(n,m,k;j)\leq2^{nmk}$ (this gives the exact amount when $j\geq nmk$). $\endgroup$ Oct 4, 2014 at 21:00
  • $\begingroup$ I am actually more interested in the tighter upper bound.... and I hope that given j, the upper bound could be polynomial (with smallest possible order) in terms of n,m,k :) thank you $\endgroup$
    – user40780
    Oct 4, 2014 at 21:28
  • 1
    $\begingroup$ If you make the matrix at random (each entry independently 0 or 1 with equal probability), I'm pretty sure that the 1s will be connected with probability ${}\to 1$ as $nmk\to\infty$. So $2^{mnk}$ is close to the correct value. To get a more interesting situation, specify that the number of 1s must be small. $\endgroup$ Oct 5, 2014 at 0:32
  • 1
    $\begingroup$ Oh, I see I misunderstood "connected". You mean only adjacent squares (please confirm), I was referring to being in the same row or column. In that case there are usually very many components. $\endgroup$ Oct 5, 2014 at 0:49
  • 2
    $\begingroup$ Possibly related MO question: "Expected number of components with multiple cycles in a subgraph of a square lattice." $\endgroup$ Oct 5, 2014 at 1:31

1 Answer 1

2
$\begingroup$

This answer has two main points:

  1. If you find a good upper bound in the case $j=1$, you will get a reasonable bound for small $j$.
  2. You cannot get polynomial bounds with respect to $n,m,k$ for any $j\geq1$. (The hope for polynomial bounds was expressed in a comment.)

Let $N(n,m,k;j)$ denote the number of choices on the $n\times m\times k$ block producing at most $j$ connected components. If we make $i$ choices with a single connected component and put them together (take the pointwise maximum if you regard each choice as a function taking values in $\{0,1\}$), we get a situation with at most $i$ connected components. This construction yields at most $N(n,m,k;1)^i/i!$ choices for each $i$, so we get the estimate $$ N(n,m,k;j) \leq \sum_{i=0}^j \frac1{i!} N(n,m,k;1)^i. $$ (Actually $N(n,m,k;1)-1$ would also work since it is the amount of choices with exactly one component, but this difference is very small.) My intuition is that this estimate is fairly good when $j$ is very small. For large $j$ it is not very tight, since $N(n,m,k;\infty)=2^{nmk}$ but the estimate gives $e^{N(n,m,k;1)}$ which is way bigger. Thus to estimate $N(n,m,k;j)$ for small $j$ the key thing is to estimate $N(n,m,k;1)$ well.

You cannot get upper bounds for $N(n,m,k;1)$ which are polynomial in $n,m,k$. Let me write $[a,b]=\{a,a+1,\dots,b\}$ for integers $a<b$. Take any function $f:[1,m]\times[1,k]\to[1,n]$. Assign the value $1$ to the cell $(a,b,c)\in[1,n]\times[1,m]\times[1,k]$ iff $a\leq f(b,c)$. (Imagine the graph of the function $f$.) Now there is exactly one connected component of $1$s, and different choices of $f$ give different choices for the $1$s. Therefore $N(n,m,k;j)\geq N(n,m,k;1)\geq n^{mk}$ for $j\geq1$, so there is no polynomial bound.

As I mentioned in a comment above, naive arguments give the bounds $$ \sum_{i=0}^j{nmk\choose i} \leq N(n,m,k;j) \leq 2^{nmk}. $$ (Choose only at most $j$ points or make any choice at all to get these bounds.)

As a summary, you have these bounds when $j\geq1$: $$ \max\left\{\sum_{i=0}^j{nmk\choose i},n^{mk},m^{nk},k^{nm}\right\} \leq N(n,m,k;j) \leq \min\left\{2^{nmk},\sum_{i=0}^j \frac1{i!} N(n,m,k;1)^i\right\}. $$ These bounds are not optimal, but give an idea of the growth rate.

$\endgroup$
5
  • $\begingroup$ great! Thank you:) For the material science problem I am facing, I will try to define a tighter way than connectedness to create the bound... $\endgroup$
    – user40780
    Oct 5, 2014 at 16:05
  • $\begingroup$ If you are interested, we could have further collaborations to write some papers:) $\endgroup$
    – user40780
    Oct 5, 2014 at 16:12
  • $\begingroup$ You are welcome! If you want to discuss possible collaboration, send email. There is a link in my profile to my homepage. $\endgroup$ Oct 5, 2014 at 16:16
  • $\begingroup$ Sure:) I will contact you when I have other ideas:) Thank you $\endgroup$
    – user40780
    Oct 5, 2014 at 16:20
  • 1
    $\begingroup$ One may give a better lower bound for $N(n,m,k;1)$ if $n$, $m$, and $k$ are large enough. Namely, one may fill all the `planes' with even first coordinate and choose an arbitrary nonempty subset in any plane with odd first coordinate. This way you obtain $(2^{mk}-1)^{\lceil n/2\rceil}$ connected sets. $\endgroup$ Oct 24, 2014 at 11:56

Your Answer

By clicking “Post Your Answer”, you agree to our terms of service and acknowledge you have read our privacy policy.

Not the answer you're looking for? Browse other questions tagged or ask your own question.